LSAT and Law School Admissions Forum

Get expert LSAT preparation and law school admissions advice from PowerScore Test Preparation.

User avatar
 Dave Killoran
PowerScore Staff
  • PowerScore Staff
  • Posts: 5852
  • Joined: Mar 25, 2011
|
#43444
Complete Question Explanation
(The complete setup for this game can be found here: lsat/viewtopic.php?t=16406)

The correct answer choice is (C)

The question stem forms a FJH super-block. This block is obviously quite space-consuming, and the space restriction should be your starting point. If the FJH block is in group 1, then L must also be in group 1 from the third rule, which is impossible due to space restrictions (again, note how important it is to show the G/M dual option on the diagram). Thus, the FJH block must be in group 2, and the other three variables—K, L, and N—must be in group 1. Consequently, answer choice (C) is correct.
 rameday
  • Posts: 94
  • Joined: May 07, 2014
|
#15530
For this game i made the inference that if

N2 :arrow: G1 :arrow: M2
and if
G2 :arrow: N1 :arrow: M1

is that correct?

I had a tough time eliminating answer choice A. I saw that FJH have to be in G2 and either G/M will have to join it.

Based on the N2 :arrow: G1 :arrow: M2 I saw that N has to be at G1

From there I think I thought If G2 :arrow: N1 :arrow: M1 but that also means K1 and A & C will both be right.

In retrospect I think I should have seen that I could have reduced the G2 :arrow: N1 :arrow: M1 to G2 :arrow: M1 and the CP would be M2 :arrow: G1 and therefore K would be in G1 but that would mae C & E correct and based on both K is always in G1 so therfore C MBT.

I feel like I made this question way to complicated!!

I also have this habit of eliminating floater variables like K that aren't restricted by rules when faced with MBT questions so I eliminated C way to prematurely.

A
 Lucas Moreau
PowerScore Staff
  • PowerScore Staff
  • Posts: 216
  • Joined: Dec 13, 2012
|
#15561
Hello, rameday,

You allllllmost made the proper inference here, lol. Very close, don't worry! It is true that N2 :arrow: G1 :arrow: M2, because N2 :arrow: G1 and G1 :arrow: M2.

However, it is not quite accurate to say that G2 :arrow: N1 :arrow: M1, because N1 does not necessarily lead to M1. It would be more proper to say G2 :arrow: N1 and M1.

You did overcomplicate things a little bit, lol. But it's fine. :lol: Since these are Must Be True answer choices, each of the incorrect ones Could Be False. It might help to try and disprove answer choices - that is, try solutions that are not what the answer choice says, and see if they're valid!

For instance, with answer choice A, we could try having G1 instead of G2. If a solution exists for G1, then answer choice A would have to be wrong. And one does!

N K L G
F H J M

Your habit of eliminating floater variables is a useful one for questions like this, mind. Same for eliminating more fixed variables in Could Be True questions. It won't always work, the way it didn't work here, but it will work enough of the time that it's always worth at least trying.

Hope that helps,
Lucas Moreau

Get the most out of your LSAT Prep Plus subscription.

Analyze and track your performance with our Testing and Analytics Package.